równanie, jaki błąd popełniam?

Własności funkcji trygonometrycznych i cyklometrycznych. Tożsamości. RÓWNANIA I NIERÓWNOŚCI.
Awatar użytkownika
elcia
Użytkownik
Użytkownik
Posty: 192
Rejestracja: 13 sty 2008, o 11:32
Płeć: Kobieta
Lokalizacja: ;)
Podziękował: 61 razy
Pomógł: 1 raz

równanie, jaki błąd popełniam?

Post autor: elcia »

Mam do rozwiązania równanie, podaje niżej z moim rozwiązaniem i tutaj wyłania się moje pytanie: jak zaznaczać na osi \(\displaystyle{ x _{1} x _{2}}\).. (jeśli jest więcej rozwiązań).
W kilku przykładasz zaznaczam na osi te x-sy i nie zgadzają mi się one z kolejnością w odpowiedziach, więc mam nadzieje, że poprawicie mi je, na rysunku na jakikolwiek inny kolor.

więc tak:
\(\displaystyle{ 3tg ^{2}x + 2 \sqrt{3} tgx -3=0}\)
\(\displaystyle{ tgx=t}\)
następnie wyliczam z delty:
\(\displaystyle{ t _{1}=- \frac{ \sqrt{3} }{3}}\)
\(\displaystyle{ t _{2} = \sqrt{3}}\)

Zaznaczam na rysunku (link),


więc wychodzi mi:
\(\displaystyle{ x _{1= \frac{PI}{3} } +kPI}\)
\(\displaystyle{ x _{2} =- \frac{PI}{6} +kPI}\)
\(\displaystyle{ k C}\)

a powinno wyjść:
\(\displaystyle{ x _{1}= - \frac{PI}{3} +KPI}\)
\(\displaystyle{ x _{2} = \frac{PI}{6} +KPI}\)
piasek101
Użytkownik
Użytkownik
Posty: 23495
Rejestracja: 8 kwie 2008, o 22:04
Płeć: Mężczyzna
Lokalizacja: piaski
Podziękował: 1 raz
Pomógł: 3264 razy

równanie, jaki błąd popełniam?

Post autor: piasek101 »

elcia pisze: następnie wyliczam z delty:
\(\displaystyle{ t _{1}=- \frac{ \sqrt{3} }{3}}\)
\(\displaystyle{ t _{2} = \sqrt{3}}\)
Mam inne znaki.

Ps. \(\displaystyle{ \pi}\)=pi
Awatar użytkownika
elcia
Użytkownik
Użytkownik
Posty: 192
Rejestracja: 13 sty 2008, o 11:32
Płeć: Kobieta
Lokalizacja: ;)
Podziękował: 61 razy
Pomógł: 1 raz

równanie, jaki błąd popełniam?

Post autor: elcia »

Faktycznie, zbyt szybko liczyłam i zagapiłam się.
A ten przykład:
\(\displaystyle{ -2sin ^{2} x+7cosx+5=0}\)
podstawiam 'jedynkę', wyliczam i wprowadzam zmienną, więc:
\(\displaystyle{ 2t ^{2} +7t+3=0}\)
delta=25
\(\displaystyle{ t _{1} =-3}\) sprzeczne
\(\displaystyle{ t _{2} =- \frac{1}{2}}\)

Mój rysunek:


więc:
\(\displaystyle{ x _{1} = \frac{2 \pi}{3} + 2k \pi}\)
\(\displaystyle{ x _{2} = - \frac{2 \pi}{3} +2k \pi}\)

Nie wiem, czy to ma jakieś znaczenie, iż na moim rysunku jest x1 i go liczę to wychodzi mi to, co ma wyść w x2? Jest jakaś kolejność opisywania osi x-mi?
piasek101
Użytkownik
Użytkownik
Posty: 23495
Rejestracja: 8 kwie 2008, o 22:04
Płeć: Mężczyzna
Lokalizacja: piaski
Podziękował: 1 raz
Pomógł: 3264 razy

równanie, jaki błąd popełniam?

Post autor: piasek101 »

elcia pisze:...
więc:
\(\displaystyle{ x _{1} = \frac{2 \pi}{3} + 2k \pi}\)
\(\displaystyle{ x _{2} = - \frac{2 \pi}{3} +2k \pi}\)
Nie wiem, czy to ma jakieś znaczenie, iż na moim rysunku jest x1 i go liczę to wychodzi mi to, co ma wyść w x2? Jest jakaś kolejność opisywania osi x-mi?
Nie sprawdzałem Twoich obliczeń.
Otrzymałaś dwie ,,serie" rozwiązań, zatem :
Odp:
\(\displaystyle{ x = \frac{2 \pi}{3} + 2k \pi}\) lub
\(\displaystyle{ x = - \frac{2 \pi}{3} +2k \pi}\) (o żadnej kolejności wyników nie ma mowy).
Awatar użytkownika
elcia
Użytkownik
Użytkownik
Posty: 192
Rejestracja: 13 sty 2008, o 11:32
Płeć: Kobieta
Lokalizacja: ;)
Podziękował: 61 razy
Pomógł: 1 raz

równanie, jaki błąd popełniam?

Post autor: elcia »

Mam jeszcze kłopot z tym:
\(\displaystyle{ cos(3x- \frac{\pi}{4} )=- \frac{ \sqrt{2} }{2}}\)
wprowadzam zmienną, więc:
\(\displaystyle{ cost=- \frac{\pi}{4}}\)
\(\displaystyle{ t _{1} = \frac{3 \pi}{4}}\)
po podstawieniu t i obliczeniu
\(\displaystyle{ x _{1} = \frac{\pi}{3} + \frac{2k \pi}{3}}\)
ten wynik mi się zgadza ale drugi nie,
ja liczę według tej zasady, że cos drugiego ma wartość równą -x bo jest symetryczny względem osi y, z tych wyliczeń wychodzi mi: \(\displaystyle{ - \frac{\pi}{6} + \frac{2k \pi}{3}}\)
piasek101
Użytkownik
Użytkownik
Posty: 23495
Rejestracja: 8 kwie 2008, o 22:04
Płeć: Mężczyzna
Lokalizacja: piaski
Podziękował: 1 raz
Pomógł: 3264 razy

równanie, jaki błąd popełniam?

Post autor: piasek101 »

elcia pisze:...
wprowadzam zmienną, więc:
\(\displaystyle{ cost=- \frac{\pi}{4}}\)
\(\displaystyle{ t _{1} = \frac{3 \pi}{4}}\)
Oczywiście możesz wprowadzać zmienną (niczego to nie ułatwia - ale może to rzecz gustu), wolę tego unikać.

Dziwnie jakoś rozwiązałaś to równanie ze względu na t (poprzednie robiłaś z rysunku, a tu w ,,pamięci" i kiszka) .

Przecież :
\(\displaystyle{ cos t = -0,5\sqrt 2}\) czyli \(\displaystyle{ t = 3x-0,25\pi}\)

\(\displaystyle{ t=0,75\pi+2k\pi}\) lub \(\displaystyle{ t=-0,75\pi+2k\pi}\) (i wracać do podstawienia).
Awatar użytkownika
elcia
Użytkownik
Użytkownik
Posty: 192
Rejestracja: 13 sty 2008, o 11:32
Płeć: Kobieta
Lokalizacja: ;)
Podziękował: 61 razy
Pomógł: 1 raz

równanie, jaki błąd popełniam?

Post autor: elcia »

już wiem, po prostu w odpowieziach jest x2 liczony po tej samej stronie
ODPOWIEDZ